The foreign minister of Zeria announced today that her country was severing diplomatic relations with Nandalo because...

Nathan on November 14, 2013

I missed this one.

I had difficulty identifying the conditions in this problem.

Replies
Create a free account to read and take part in forum discussions.

Already have an account? log in

Naz on November 19, 2013

The foreign minister of Zeria decided to sever diplomatic relations with Nandalo because of Nandalo's flagrant violations of human rights. The argument then concludes that this diplomatic move cannot be explained fully by Zeria's commitment to upholding human rights because Zeria still maintains diplomatic relations with many countries that have far worse human-rights records than Nandalo.

We are asked to choose the argumentative structure that most closely parallels that of the argument in the passage.

(A) is incorrect because to parallel the passage above, the reasoning given for the conclusion should be an example where Henry's parents were not concerned for Henry's health. However, the counter evidence given shows an instance where Henry's parents are not concerned for their own health. Remember, in the passage we conclude that despite an action showing Zeria's commitment to upholding human rights (their severing of diplomatic relations with Nandalo), due to counter evidence showing that Zeria still maintains diplomatic relations with many countries that have far worse human-rights records than Nandalo, the recent severing of diplomatic relations with Nandalo must not have been exclusively due to this commitment. Thus, to have similar reasoning, this answer choice should have offered counter evidence showing that Henry's parents were not concerned with HIS health, rather than their own.

(B) is incorrect because there is no counter evidence given that shows that Professor Walsh's policy against handwritten term papers due to most handwriting being difficult to read is inconsistent. We are told that she lectures from handwritten notes, however, this does not suffice as counter evidence to her policy because her handwriting her own notes does not show an inconsistency with her not being able to read term papers that are handwritten by others. Term papers are longer than notes, and her policy is for her students as opposed to herself. Thus, the reasoning in this answer choice does not parallel that of the passage above.

(C) is incorrect because it does not follow the same argumentative structure as the passage above. James claims that he stole only because he was hungry. We therefore need counter evidence that cites an example of a time that James stole when he was not hungry. No such counter evidence is given to us.

(D) is CORRECT because it follows the same argumentative structure as the passage above. Armand declined Helen's invitation to dinner because he believes that socializing with coworkers is imprudent. We are then given counter evidence that Armand actually went to a movie with another coworker that same evening. Thus, his concern for prudence cannot be fully explained his refusal to go to dinner with Helen. Just like the passage, we are given someone's action and the reason/belief behind that action. Then, we are given counter evidence that weakens the reasoning behind the action. And we conclude that the reason given was not completely explicative of why the action took place.

(E) is incorrect because the counter evidence given to us does not go against the reasoning in the answer choice. The answer choice claims that the reason why there are fewer good teachers than there used to be is because salaries have reached a new low. Then the answer choice cites that this reason does not fully explain why there are fewer good teachers because teachers have always been poorly paid. But, even if teachers have always been poorly paid, it still could be true that recently their salaries have been at an all time low, which could fully explain why there are fewer good teachers than there used to be. Thus, the argument structure of this answer choice does not fully parallel that of the passage above.

Hope that was helpful! Let me know if you have any other questions.

Gabe85 on July 23, 2020

I struggled with this question but this really helped, thank you!